Proving the Sum of Cubes Formula using Mathematical Induction

  • Thread starter Thread starter efekwulsemmay
  • Start date Start date
  • Tags Tags
    Induction Proof
Click For Summary
The discussion centers on proving the formula for the sum of cubes using mathematical induction. The user has successfully shown that the formula holds for n=1 and is now attempting to prove it for n=p+1, starting from the assumption that it holds for n=p. They have reached a point where they need to show that the sum of cubes up to p+1 can be expressed in terms of the formula. Participants suggest expanding and simplifying the expression to complete the induction step. The conversation emphasizes the importance of algebraic manipulation in the proof process.
efekwulsemmay
Messages
53
Reaction score
0
Nevermind, got the answer :D

Homework Statement


I need to prove using mathematical induction that:

\sum^{n}_{i=1} i^{3}= 1^{3}+2^{3}+3^{3}+...+n^{3}=\left[\frac{(n+1)\cdot n}{2}\right]^{2}


Homework Equations



\sum^{n}_{i=1} i^{3}= 1^{3}+2^{3}+3^{3}+...+n^{3}=\left[\frac{(n+1)\cdot n}{2}\right]^{2}


The Attempt at a Solution



Ok, I have proved that the statement is true when n=1 and have gone on to assume that it must be true for any arbitrary value of n. I chose n=p. Thus:

\sum^{p}_{i=1} i^{3}= 1^{3}+2^{3}+3^{3}+...+p ^{3}=\left[\frac{(p+1)\cdot p}{2}\right]^{2}

Where i am stuck is that I know i need to show that if the statement is true when n=p then it must be true for n=p+1. I have gotten as far as:

\sum^{p}_{i=1} i^{3}= 1^{3}+2^{3}+3^{3}+...+p^{3}+(p+1)^{3}=\left[\frac{(p+1)\cdot p}{2}\right]^{2}+(p+1)^{3}
 
Last edited:
Physics news on Phys.org
<br /> \sum^{p}_{i=1} i^{3}= 1^{3}+2^{3}+3^{3}+...+p^{3}+(p+1)^{3}=\left[\frac{(p+1)\cdot p}{2}\right]^{2}+(p+1)^{3}<br />

Factor out a (p+1)^2, and see what you see.
 


efekwulsemmay said:

Homework Statement


I need to prove using mathematical induction that:

\sum^{n}_{i=1} i^{3}= 1^{3}+2^{3}+3^{3}+...+n^{3}=\left[\frac{(n+1)\cdot n}{2}\right]^{2}


Homework Equations



\sum^{n}_{i=1} i^{3}= 1^{3}+2^{3}+3^{3}+...+n^{3}=\left[\frac{(n+1)\cdot n}{2}\right]^{2}


The Attempt at a Solution



Ok, I have proved that the statement is true when n=1 and have gone on to assume that it must be true for any arbitrary value of n. I chose n=p. Thus:

\sum^{p}_{i=1} i^{3}= 1^{3}+2^{3}+3^{3}+...+p ^{3}=\left[\frac{(p+1)\cdot p}{2}\right]^{2}

Where i am stuck is that I know i need to show that if the statement is true when n=p then it must be true for n=p+1. I have gotten as far as:

\sum^{p}_{i=1} i^{3}= 1^{3}+2^{3}+3^{3}+...+p^{3}+(p+1)^{3}=\left[\frac{(p+1)\cdot p}{2}\right]^{2}+(p+1)^{3}

have you tried just expanding it out and seeing what you get?
the induction step is simple, you got that - the rest is just algebra
 
Basically, you should expand, collect terms, and reduce. You should be able to do it that way.
 
Question: A clock's minute hand has length 4 and its hour hand has length 3. What is the distance between the tips at the moment when it is increasing most rapidly?(Putnam Exam Question) Answer: Making assumption that both the hands moves at constant angular velocities, the answer is ## \sqrt{7} .## But don't you think this assumption is somewhat doubtful and wrong?

Similar threads

  • · Replies 7 ·
Replies
7
Views
2K
  • · Replies 7 ·
Replies
7
Views
2K
  • · Replies 12 ·
Replies
12
Views
3K
  • · Replies 22 ·
Replies
22
Views
966
  • · Replies 3 ·
Replies
3
Views
2K
  • · Replies 7 ·
Replies
7
Views
1K
  • · Replies 4 ·
Replies
4
Views
1K
  • · Replies 1 ·
Replies
1
Views
2K
Replies
4
Views
2K
  • · Replies 6 ·
Replies
6
Views
2K